Diskussion:Bellsche Ungleichung

aus Wikipedia, der freien Enzyklopädie
Letzter Kommentar: vor 16 Tagen von Qcomp in Abschnitt Noch einfachere Herleitung
Zur Navigation springen Zur Suche springen
Zum Archiv

Anforderungen an das Experiment[Quelltext bearbeiten]

Dass es (Punkt 2) keine überlichtschnellen Signale gibt unter "Anforderung an das Experiment" aufzuführen ist unglücklich formuliert. Und was heisst bei Punkt 3, (erst) bei Rowe wäre das Nachweisschlupfloch geschlossen worden, für die vorhergehenden Experimente, die waren doch wohl dadurch, dass einige Photonen nicht nachgewiesen wurden, bezüglich des Nachweises der Verletzung der Bellungleichung nicht fehlerhaft ? Die Annahme, die diesem Schlupfloch zugrunde liegt, dass nicht detektierte Photonen sich anders verhalten als detektierte so dass die Bell-Ungleichung in der Summe doch noch erfüllt ist, ist ziemlich exotisch. "Überzeugend" war der Nachweis der Verletzung der Bell-Ungleichungen für die meisten Physiker schon vorher.--Claude J 10:18, 22. Jun. 2010 (CEST)Beantworten

EPR-Paradoxon als historische Vorlage[Quelltext bearbeiten]

Hallo, ich fände es gut das EPR-Paradoxon bzw. Argument als Motiv für Bells Überlegungen anzugeben. Zum einen bezieht er sich in seiner Arbeit direkt darauf, zum anderen wird die Bedeutung der Bellschen Ungleichung in diesem Zusammenhang klarer. (nicht signierter Beitrag von 77.7.20.143 (Diskussion) 12:33, 18. Aug. 2010 (CEST)) Beantworten

Wahrscheinlichkeitstheoretische Sicht fehlt[Quelltext bearbeiten]

E. T. Jaynes betrachtet EPR und die Bellschen Ungleichungen aus wahrscheinlichkeitstheoretischer Sicht, siehe http://www.cim.mcgill.ca/~mgarden/pubs/jaynes-mystery.pdf . Aus seiner Sicht ist der Bellsche Ansatz eine durch kein Experiment gerechtfertigte Einschränkung des allgemeinen wahrscheinlichkeitstheoretischen Ansatzes. Da das quantenmechanische Resultat nicht in dieser Bellschen Weise faktorisierbar ist, widerspricht es für gewisse Winkel den Bellschen Ungleichungen. Allerdings widerspricht das quantenmechanische Resultat keineswegs dem korrekten wahrscheinlichkeitstheoretischen Ansatz à la Jaynes.

Bells Ansatz wird kritisiert, weil er die Lokalität der beiden Teilchen in der Raunzeit (ein ontologischer Standpunkt) in seinem Ansatz "gleich setzt" mit der Unabhängigkeit der Wahrscheinlichkeitsverteilungen (ein epistemologischer Standpunkt). Es ist aber seit Boole bekannt, dass man aus den Wahrscheinlichkeitsverteilungen nicht auf die Dynamik in der Raunzeit schließen kann. Hier gibt Jaynes ein sehr einfaches klassisches Beispiel im o.g. Artikel. Ein weiteres Beispiel geben Hess, Michielsen und De Raedt in K. Hess, K. Michielsen, and H. De Raedt, Possible Experience: from Boole to Bell, Europhys. Lett. 87, 60007 (2009), http://iopscience.iop.org/0295-5075/87/6/60007/pdf/0295-5075_87_6_60007.pdf .

Akzeptiert man Jaynes Standpunkt (der wohlgemerkt grundlegender ist als der von Bell und aus welchem man Bells Ansatz nur durch logisch nicht begründbare ad-hoc-Einschränkungen gewinnen kann), dann sind die Bellschen Ungleichungen als Kriterium für räumliche "Lokalität" nicht relevant, weil der Bellsche Ansatz von dem Mißverständnis ausgeht, dass Lokalität in der Raumzeit die Separabilität der Wahrscheinlichkeitsverteilungen impliziert. D.h. aber auch, dass die entsprechenden Experimente keine Aussage zur Lokalität in der Raumzeit machen können. Damit wäre aber schlußendlich die Aussage des Artikels nicht gerechtfertigt, dass "dass nicht alle Messwerte vor der Messung feststehen oder dass die Meßwerte nichtlokal von weit entfernten, zufälligen Entscheidungen abhängen...".

M.E. muss diese in weiten Kreisen der einschlägigen Community wohlbekannte Kritik im Artikel Berücksichtigung finden.

-- Rhymer 12:12, 20. Mär. 2011 (CET)Beantworten

Versuchsaufbau[Quelltext bearbeiten]

Mir fehlt im Artikel eine kurze Beschreibung des Versuchsaufbaus zur Herleitung. Bei diesem Satz: "Entsprechend sind oder , je nachdem ob das zweite Photon des Paares im Versuch mit der Nummer durch den zweiten, in Richtung polarisierten Filter kommt oder nicht." stellt sich mir die Frage wie ich mir eine zweite Messung am zweiten Photon vorstellen soll. Pumuckl2 (Diskussion) 10:12, 11. Mai 2012 (CEST) (interessierter Laie)Beantworten

Realismus und Lokalität[Quelltext bearbeiten]

Da es zu meinem Beitrag zu diesem Thema vom Juli d.J. keinen Widerspruch gegeben hat, aber auch keine Korrektur im Artikel, habe ich das jetzt mal getan, soweit es ohne neue Quellen geht.

Zum Realismus ist anzumerken, dass die verborgenen Variablen eigentlich nicht in die Definition gehören. Sie waren nur ein Versuch, ein realistisches Verständnis der Quantenmechanik zu erlauben. Da sie für den Artikel wichtig sind, habe ich sie als Beispiel in dem Satz gelassen, allerdings in den Satzteil verlegt, in den sie gehören.

Bei der Lokalität schien mir besser (und schon grammatisch einfacher), etwas über Nichtlokalität als über Lokalität zu sagen. Durch die Worttrennung ‚nicht lokal‘ statt ‚nichtlokal‘ ist jetzt aber nicht mehr behauptet, dass das eine Definition sei. Es ist nur noch eine mögliche Ausprägung von Nichtlokalität. Die formulierte Aussage dürfte unstrittig sein und genügt im Rahmen des Artikels.-- Binse (Diskussion) 17:37, 8. Jan. 2018 (CET)Beantworten

Diese Ungleichung drückt nun WAS genau aus?[Quelltext bearbeiten]

Im Artikel:

1.Begriffskläung 2.Herleitung

Eine Herleitung ohne zu erkären was man da überhaupt herleitet?


Bitte.

1 .Erst die Ungleichung 2. Dann die Interpretation 3. Dann wie jetzt 1. Begriffserklärung

...


mfg (nicht signierter Beitrag von 88.150.93.29 (Diskussion) )

Was hergeleitet wird steht gleich in der Einleitung, eine Ungleichung für die Korrelationen von Messergebnissen, die in der QM verletzt ist. Danach kann man nicht gleich die Ungleichung, die im Abschnitt Herleitung steht, hinknallen, sondern muss das im Rahmen der Herleitung erläutern. Die meisten Leser dürfte die genaue Formulierung als technische Einzelheit auch gar nicht interessieren, sondern eher die Schlussfolgerungen im Rahmen der Interpretation der QM, die im zweiten Abschnitt vorangestellt ist.--Claude J (Diskussion) 19:55, 2. Feb. 2018 (CET)Beantworten

Verletzung in Quantenmechanik[Quelltext bearbeiten]

Die Ungleichung kann durchaus auch in der Quantenmechanik gelten, es hängt von der Wahl der Richtungen a,b,c ab (siehe die Formel unten im Abschnitt QM). Insofern ist das was jetzt in der Zusammenfassung steht genaugenommen falsch.--Claude J (Diskussion) 23:25, 22. Apr. 2018 (CEST)Beantworten

Das wäre an sich leicht zu korrigieren. Man muss nur etwas präziser sagen: Für die Q. gilt die B.U. nicht immer. Es steht aber so viel Anfechtbares in der Zusammenfassung, dass ich dabei bleibe, sie als Ganzes zu ersetzen. Nach den Regeln der WP könnt ihr sie ja dann wieder bearbeiten.
Die im Artikel bewiesene und in dieser oder einer anderen Formulierung auf Bell zurückgehende Ungleichung ist hier unter Verwendung von Korrelationskoeffizienten geschrieben, nicht mit dem verlinkten Kontingenzkoeffizienten. Mit diesem haben sie kaum etwas zu tun. Bells Ungleichung bedeutet auch keine Obergrenze für die drei darin enthaltenen Korrelationskoeffizienten: Wenn alle drei =1 sind, also so groß wie möglich, die Korrelation so stark wie möglich, ist die Ungleichung doch erfüllt! Da einer dieser Koeffizienten mit negativem Vorzeichen in der Ungleichung steht, ist auch völlig unklar, was die in der Quantentheorie stärkere Korrelation überhaupt bedeuten soll. Diese Behauptung ist, zumindest ohneBegründung, schlicht sinnlos. Da das Aggregat aus Korrelationskoeffizienten mit unterschiedlichen Vorzeichen nichts zum Verständnis der Ungleichung beiträgt, bevorzuge ich eine andere Formulierung, die sich z.B. bei Zeilinger[1] findet. Es ist hier nicht der Platz, darauf einzugehen. Es betrifft ja die Durchführung.
Die Zusammenfassung sagt, die U. beträfe Spin-Variablen. Das ist vermutlich korrekt als Zitat auf Bell. In der Durchführung werden aber lineare Polarisationen betrachtet, wie in vielen der bekannten Experimente. Hat nicht die Zusammenfassung die Durchführung zusammenzufassen? Dass die Q. für verschränkte Teilchen etwas Anderes voraussagt, sieht so aus, als wäre das bei Bell ein Spezialfall. Tatsächlich geht es doch wohl auch bei Bell von vornherein um verschränkte Systeme. Der (unausgesprochene) Ausgangspunkt, der auch in der Durchführung des WP-Artikels gebraucht wird, ist doch, dass auf einen Quantenzufall hier zwei Messungen kommen. Bei nicht verschränkten Teilchen sehe ich keinen vergleichbaren Aufhänger.
Dass die Verletzung der B.U. schon 1935 paradox erschienen sein soll, ist ein Anachronismus, natürlich nicht so gemeint, und müsste umformuliert werden. Der Satz gehört aber sowieso entfernt, denn die Zusammenfassung zur B.U. ist nicht der Ort für so eine Bemerkung.
Die Worte überlichtschnelle Wirkung würde ich ersetzen durch nichtlokale Korrelationen. Das ist, was gemeint ist, und weckt keine falschen Assoziationen. Ich lasse aber den ganzen Satz erst mal weg, weil es zur lokalen Kausalität keinen Link gibt und mir der Unterschied zum lokalen Realismus aus der beigefügten Erklärung nicht klar wird. Welche zusätzliche Denkmöglichkeit bietet diese Abschwächung? Vielleicht will Reinald62 das wieder anfügen, oder Realismus passend ergänzen.-- Binse (Diskussion) 13:15, 27. Apr. 2018 (CEST)Beantworten

beweis rechnung wiederlegt nicht die sache[Quelltext bearbeiten]

also die bellsche ungleichung ausgerechnet ergibt bei mir genau das was einstein rosen und podolski these ist. also die wahrscheinlichkeit das es rechts geblockt und links durchgeht W(b_h,c_v)= cos(30)^2 nicht sin. es ist ein ding mit versteckten variablen, die kommen an beiden orten an. also das einsetzen erfuellt die ungleichung. viele grueße (nicht signierter Beitrag von Alex fdhsjrtfg82 (Diskussion | Beiträge) 05:49, 11. Nov. 2019 (CET))Beantworten

Entschwurbelung[Quelltext bearbeiten]

Schon die ganze Einleitung und der erste Abschnitt Realismus und Lokalität sind total verschwurbelt. Man müßte ganz einfach schreiben, daß die Bellsche Ungleichung nichts anderes als eine Definition ist, wann eine Theorie klassisch ist und wann nicht. Wenn die Bellsche Ungleichung: "verletzt" wird, bedeutet das einfach nur, daß eine Theorie nichtklassisch ist. Dabei gibt Bell die zwei Kriterien Realismus und Lokalität an.

  • Realismus bedeutet, ob Messungen für verläßlich gehalten werden oder grundsätzlich für den Arsch sind (erster Fall: Klassische Theorie, zweiter Fall: Kopenhagener Interpretation).
  • Und Lokalität bedeutet im Grunde, ob an Quantenverschränkung geglaubt wird (wenn nein: Klassische Theorie, wenn ja: Kopenhagener Interpretation).

Ob es Quantenverschränkung gibt, kann man dabei schon aus zwei Gründen heraus nicht empirisch beweisen: Erstens, weil man unter realen Bedingungen nicht schnell genug über kosmische Entfernungen messen kann, zweitens aufgrund des grundsätzlichen Irrealismus der die Verschränkung postulierenden Kopenhagener Interpretation, wonach Messungen ohnehin grundsätzlich für'n Arsch sind. Einsteins klassische Handschuhmetapher eignet sich weitaus besser zur Erklärung der empirisch beobachtbaren Phänomene als das Konzept der Verschränkung. --2003:DA:CF12:2F00:41FE:ADA0:5D24:C129 15:02, 22. Aug. 2020 (CEST)Beantworten

Einleitungssatz? Bellsche Ungleichung betrifft nicht "verschränkte Paare"[Quelltext bearbeiten]

Die Bellsche Ungleichung betrifft nicht verschränkte Systeme: sie ist eine Aussage über die Korrelationen, die in lokal-realistische Systemen beobachtet werden koennen, in denen es gerade keine Quantenverschränkung gibt. Sie ist interessant, weil sie im Widerspruch zu dem steht, was die QM für die Korrelationen der Ergebniss für bestimmte Messungen an bestimmten verschränkten Zuständen vorhersagt. Ich finde, der Satz sollte besser lauten:

Die Bellsche Ungleichung betrifft Messreihen an Teilchenpaaren. Sie wurde 1964 von John Stewart Bell veröffentlicht, um ein Konzept Einsteins zu analysieren. Sie formuliert einen experimentell überprüfbaren Widerspruch zwischen den Konsequenzen des Einsteinschen Konzepts und den Vorhersagen der Quantenmechanik. Zahlreiche Experimente haben seither die Verletzung der Ungleichung nachgewiesen und damit die Vorhersagen der Quantenmechanik bestätigt.

Eigentlich würde ich statt "Konzept Einsteins" lieber "lokaler Realismus" schreiben, aber ich vermute, das wurde schon gründlich diskutiert und mir geht es jetzt darum, den Bezug zur Verschränkung klarzustellen, der mE im derzeitigen Text etwas irreführend ist. Bitte um Kommentare.--Qcomp (Diskussion) 20:20, 24. Jun. 2021 (CEST)Beantworten

Sollte die Einleitung nicht mit einem etwas allgemeineren Satz beginnen als mit einem eher technischen Hinweis auf das Einsatzgebiet? Vielleicht etwa so?
Die B.U. formuliert eine Bedingung, mit der sich nachweisen lässt, dass die für die Quantenphysik typische Unbestimmtheit vieler Messergebnisse nicht grundsätzlich auf die Unkenntnis weiterer, verborgener Variabeln zurückgeführt werden kann. (...)
Für OMA, glaube ich, ist das interessanter.--Bleckneuhaus (Diskussion) 23:37, 23. Mär. 2024 (CET)Beantworten

Ungrammatischer Satz?[Quelltext bearbeiten]

Unter "Realismus und Lokalität" steht unter 2.

Eine physikalische Theorie ist nicht lokal, im Sinne der speziellen Relativitätstheorie, wenn in raumartiger Relation die Messergebnisse an zwei Teilchen korreliert sind, eine dem Zufall widersprechende Beziehung zeigen, ...

Der Gliedsatz ", wenn ..." endet mit "korreliert sind,". Was ist aber das Segment "eine ... zeigen"? Ein weiterer Gliedsatz (dann fehlt die Konjunktion oder das Relativpronomen)? Ein gleichgeordneter Satzteil mit einem fehlenden "das heißt"? Der Satz ist m.E. grammatikalisch falsch und damit unverständlich - aber vielleicht blicke ich nur nicht durch ... --Haraldmmueller (Diskussion) 08:09, 30. Nov. 2023 (CET)Beantworten

Ja, so sehe ich das auch. Evtl. ist gemeint: "Eine physikalische Theorie ist nicht lokal, im Sinne der speziellen Relativitätstheorie, wenn in raumartiger Relation die Messergebnisse, die an zwei Teilchen korreliert sind, eine dem Zufall widersprechende Beziehung zeigen, ohne dass dies mit objektiv vorliegenden Eigenschaften der Teilchen erklärt werden könnte."
--Zulu55 (Diskussion) 08:52, 30. Nov. 2023 (CET)Beantworten
Wär zwar grammatisch ok - aber inhaltlich kann ich "Messergebnisse, die an zwei Teilchen korreliert sind" auch nur mit wackeliger (Um-)Deutung verstehen. Ich probier: Eine physikalische Theorie ist nicht lokal im Sinne der speziellen Relativitätstheorie, wenn Messergebnisse an zwei in raumartiger Relation voneinander entfernten Teilchen eine dem Zufall widersprechende Beziehung zeigen, ohne dass dies mit objektiv vorliegenden Eigenschaften der Teilchen erklärt werden könnte. Ja, da hab ich nun mehr geändert = ist nun Frage der physikalischen Korrektheit; aber so, denke ich, hab ich's gelernt. --Haraldmmueller (Diskussion) 11:58, 1. Dez. 2023 (CET)Beantworten

Leichtere Erklärung der Ungleichung gesucht[Quelltext bearbeiten]

An der Herleitung im Abschnitt Die Ungleichung bei Annahme von verborgenen Variablen stört mich, dass ich sie nicht einfach nachvollziehen kann. Insbesondere steige ich beim Satz "Lässt man nun eine der drei genannten und einschränkenden Eigenschaften weg, so bleiben die gefundenen Anzahlen entweder gleich oder werden größer." aus, denn welche "drei genannten Eigenschaften" mögen da gemeint sein? Im Text rein sprachlich nicht zu identifizieren.

Mir persönlich sagt auch die mengentheoretisch formulierte Erklärung deutlich mehr zu: Bell-theorem2.

Diese zusätzlich zu bringen, könnte aber wie Aufblähung wirken. wenn ich die bestehende Erklärung dadurch ersetze - würde sich Protest regen? --Bleckneuhaus (Diskussion) 17:46, 21. Mär. 2024 (CET)Beantworten

Meine obige Frage konnte ich durch angestrengtes LEsen klären, und hab den Text in dieser Hinsicht etwas klarer zu machen versucht. Beim Nachvollziehen des Arguments bin ich aber auf einen anderen verdächtigen Satz gestoßen: "Unter den insgesamt von Alice und Bob vermessenen Photonen-Zwillingen ist die Anzahl der großen blonden Zwillinge gleich der Anzahl der großen blonden Männer plus der Anzahl der großen blonden Frauen." Was, wenn es zB gar keine Zwillinge gibt, die groß und blond sind? Das scheint mir nirgends ausgeschlossen. - Die ganze Herleitung ist von @Binse: ([1]) der aber seit 2021 nichts mehr schreibt. Da ein Lit-Hinweis fehlt, auch in der damaligen Diskussion, wäre Theoriefindung denkbar, incl. der Möglichkeit, Fehler eingebaut zu haben (Verzeihung, Binse!). Weiß jemand näheres? --Bleckneuhaus (Diskussion) 00:31, 22. Mär. 2024 (CET)Beantworten


Vorschlag für einfache Darstellung (soll den vorhandenen Text ab Für einen Moment sollen anschauliche Codeworte die mathematischen Zeichen ersetzen ersetzen):



Es gebe Objekte und drei Eigenschaften , so dass jedes Objekt jede der Eigenschaften entweder hat oder nicht hat. Sei die Menge der Objekte, die sowohl als auch haben (wobei unbeachtet bleibt, ob sie haben oder nicht haben [] ). Diese Menge ist sicher in der Vereinigungsmenge von zwei anderen analog gebildeten Mengen enthalten:

Denn jedes Element von , das nicht schon im ersten Summanden enthalten ist, muss sicher die beiden Eigenschaften und haben, und liegt damit im zweiten Summanden. Für die Anzahlen (mit etc. bezeichnet), gilt also

Die beiden Grafiken (noch besser layouten) veranschaulichen diese Herleitung.

Die Objekte sind Punkte im Rechteck; die Linien darin grenzen die Gebiete ab, in denen die Objekte die betreffende Eigenschaft haben bzw. nicht haben.
Bellsche Ungleichung

.



Wenn das für das Zustandekommen der Ungleichung als solcher akzeptiert werden kann, sollte der übrige Text auch von unnötigen Aussagen befreit werden. Statt v,f,e sollte man auch liebber a,b,c o.ä. nehmen, aber ich kann die Grafiken nicht editieren. --Bleckneuhaus (Diskussion) 23:50, 22. Mär. 2024 (CET) Kl. Überarbeitung --Bleckneuhaus (Diskussion) 04:36, 23. Mär. 2024 (CET)Beantworten

Einleitung neu (für allgemeinere Leserschaft) - ein Vorschlag[Quelltext bearbeiten]

""Die bellsche Ungleichung (englisch Bell’s Theorem) ist eine Ungleichung zwischen den Korrelationen von Messergebnissen an verschränkten Quantensystemen. Mit ihrer Hilfe konnte durch Experimente nachgewiesen werden, dass die für die Quantenphysik typische Unbestimmtheit vieler Messergebnisse nicht immer durch eine unvollständige Kenntnis der Quantensysteme hervorgerufen wird, sondern naturgegeben ist.

Die Ungleichung wurde 1964 von John Stewart Bell aufgestellt,[2] um die von Albert Einstein vertretene Form des Realismus zu prüfen, nach der bei jedem physikalischen System die Eigenschaften klar festgelegt sind, unabhängig davon, was menschliche Beobachter über sie wissen oder denken.[3]

Einstein hatte 1935 zusammen mit Boris Podolsky und Nathan Rosen,[4] kurz EPR, argumentiert, dass die Quantentheorie offenbar unvollständig sei. Denn in der Physik müsse es möglich sein, den Teilchen in jedem Zustand individuelle Eigenschaften zuzuschreiben, die ihr Verhalten steuern und damit auch bei Messungen den quantenmechanischen Zufall vortäuschen.

Bell zeigte theoretisch, dass bei allen Experimenten an Teilchenpaaren die Korrelationen zwischen den Messergebnissen durch die nach ihm benannte Ungleichung beschränkt sind, falls zusätzlich zum Realismus die Lokalität gilt, nach der Signale (jede Art von Information und Kausalität) sich maximal mit Lichtgeschwindigkeit ausbreiten können. Die Gültigkeit beider Annahmen zusammengenommen wird als lokaler Realismus bezeichnet. Da die Quantenmechanik aber für Teilchenpaare in bestimmten verschränkten Zuständen die Verletzung der Ungleichung voraussagt, formuliert die Bellsche Ungleichung eine Möglichkeit, den lokalen Realismus experimentell zu überprüfen.

Was 1964 Bell als Gedankenexperiment beschrieb, konnte ab 1972 in echten Experimenten durchgeführt werden,[5] zuerst von Stuart Freedman und John Clauser.[6] Zahlreiche Experimente haben seither die Verletzung der Ungleichung für verschränkte Teilchenpaare nachgewiesen und die Vorhersagen der Quantenmechanik in jedem Fall bestätigt. Für diese Erkenntnis erhielten Alain Aspect, John Clauser und Anton Zeilinger im Jahr 2022 den Physiknobelpreis.

Aufgrund der Ergebnisse der Experimente gilt Einsteins Konzept des lokalen Realismus heute als widerlegt. Mindestens eines der beiden Prinzipien von Lokalität und Realismus muss bei der Betrachtung verschränkter Teilchenpaare aufgegeben werden.""


So weit mein Voschlag. Wenn er noch ein paar Tage abhängt, möchte ich ihn einstellen. --Bleckneuhaus (Diskussion) 00:22, 26. Mär. 2024 (CET)Beantworten

Da ich jetzt für 10 Tage ohne Computer sein werde, habe ich es heute schon getan, mit minimalen stilistischen Glättungen. --Bleckneuhaus (Diskussion) 17:28, 27. Mär. 2024 (CET)Beantworten
Sorry, ich war verreist, daher keine Zeit zum Kommentieren. (1) Die Bell-Ungleichung ist eine Ungleichung, die für lokal-realistische Systeme gilt (unabhängig davon, ob verschränkt oder nicht). Von verschränkten Quantensystemen wird sie verletzt. Daher sollte der erste Satz besser lauten:
Die bellsche Ungleichung (englisch Bell’s Theorem) ist eine Ungleichung für die Korrelationen von Messergebnissen, die von Messungen an verschränkten Quantensystemen verletzt werden kann.
(2) Was die Bell Ungleichung von anderen Belegen für den fundamentalen Indeterminismus der QM unterscheidet, ist, dass sie ihn an zwei sehr fundamentale Annahmen koppelt, nämlich die Gültigkeit von Einstein-Lokalität (keine Signalübertragung mit Überlichtgeschwindigkeit) und die Möglichkeit, Messeinstellungen unabhängig vom beobachteten System zu wählen. Eine Verletzung der Ungleichung lässt Anhängern des Realismus dann nur noch die Wahl zwischen Nichtlokalität oder Superdeterminismus. Mir fehlt dieser Aspekt (v.a. der der Lokalität; es geht ja mMn mehr um die Widerlegung des Lokalen Realismus denn um den Indeterminismus der QM) im zweiten Satz.
Durch die experimentelle Verletzung der Ungleichung konnte nachgewiesen werden, dass die Quantenphysik sich nicht durch eine lokale und realistische Theorie beschreiben lassen und dass die für die Quantenmechanik typische [Zufälligkeit|Undeterminiertheit|Unbestimmtheit] vieler Messergebnisse nicht durch genauere Kenntnis des Systems beseitigt werden kann, sondern naturgegeben ist.
(3) warum eigentlich "engl. Bell's Theorem"? "Bell inequality" ist auch im englichen geläufig (gleich viele Google-Treffer) und eigentlich sind es zwei verschiedene Dinge. Einerseits die Ungleichung, andererseits der Satz, dass alle einer lokal-realistischen Theorie genügenden Systeme die Ungleichung erfüllen.
(4) Im Absatz "Bell zeigte theoretisch..." sollte mE auch die zweite wesentliche Annahme (dass die Messeinstellungen unabhängig voneinander und vom Zustand des beobachteten teilchen gewählt werden kann) genannt werden.
(5) Im Absatz "Was 1964 Bell als..." könnte man evt ergänzen, dass die Experimente nach 1972 sich immer mehr den idealen, von Bells Theorem verlangten Bedingungen annäherten. (Damit klar ist, dass es nicht nur um Bestätigungen des ersten Experiments handelt).
(6) Der „Superdeterminismus“ ist zwar logisch ein nicht zu schliessendes Schlupfloch für die Einsteinlokalität und wäre daher uU im letzten Satz einschränkend zu nennen. Andererseits gibt es aber meines Wissens nicht einmal den Ansatz für eine plausible superdeterministische Theorie, die alle Bell-Experimente (und den Rest der QM) erklärt, sodass es mE schon korrekt ist, zu sagen, dass der lokale Realismus als (Physiker-)widerlegt gilt. Daher ist es mE ok, das im letzten Satz der EL nicht anzusprechen. --Qcomp (Diskussion) 22:38, 27. Mär. 2024 (CET)Beantworten
Ein Kilodank, Qcomp! Das kann ich glaub ich alles übernehmen - sobald ich wieder ordentlich dazu kommw. --Bleckneuhaus (Diskussion) 00:15, 28. Mär. 2024 (CET)Beantworten
Bis auf den Superdeterminismus konnte ich eben doch noch ungefähr alles mal einbauen. Der Superdeterminismus ist in meinen Augen auch eine so abwegige Annahme, dass er nicht mit in die Einleitung gehört. Besser so? --Bleckneuhaus (Diskussion) 18:18, 28. Mär. 2024 (CET)Beantworten

noch eine Herleitung einer Bellschen Ungleichung, absolut einfachst[Quelltext bearbeiten]

In dem Quassel-Blog quora gefunden: (Auszug aus [2] , und könnte noch gekürzt werden)

Imagine that Alice and Bob have a large number of boxes. Each box contains three coins, and coins can be gold or silver. Alice and Bob intend to work their way through all of the boxes. For each box Alice will reach in and draw out a coin, and then Bob will reach in and draw out a coin. Without looking, of course - the coins they wind up with will be random. Then they compare their coins and record whether their coin types match or dont’ match. At the end of the entire series of tests they will have a match/mismatch percentage.
Prior to the session, you get to prepare the boxes any way you want. Your goal is to control what match/mismatch percentage they wind up with when they’re done. Hopefully this is simple and clear to you.
Some things are easy for you to achieve. Say you want them to match 100% of the time. That’s easy - you just make every single coin gold, or every single coin silver, and then there is no way for them to mismatch. However, what if your goal is to have them match less than 100% of the time - say N% of the time. If they match N% of the time, they will mismatch 100-N % of the time, obviously.
Ok, what values of N can you achieve? There are really only two ways you can prepare a box that “matter.” You can either make all three coins identical, or you can make two of them identical and the third opposite. If Alice and Bob use a box that has three matching coins, their choices will match - there’s no way around that. Expressing that formally, we say p(match) | identical = 1.0. If, however, they choose a box with two coins the same and the third different, then there are other possibilities.
Let’s go through these. Alice has a 1/3 probabiility of selecting the odd coin. In that case they will mismatch, because Bob only has mismatching coins left to choose. On the other hand, Alice has a 2/3 probability of selecting one of the matching coins, and then there is a 50% chance of Bob selecting the other one. So, p(match) | non-identical = (2/3)*(1/2) = 1/3.
Let’s say you prepare the boxes so that M% of the boxes are identical and 100-M% are non-identical. The overall match rate Bob and Alice will achieve then has to be
Overall_Match = M + (100-M)*(1/3) = (1/3) + (2/3)*M
You can choose M to be any value from 0% to 100%. Now, consider what this means. There is no way for you to cause Alice and Bob to match less than 1/3 of the time, even if you set M to 0. This is the kind of statement that Bell’s theorem makes about classical physics.
Real tests in physics can be crafted that are described by this same kind of logic. And it turns out that things can be arranged so that quantum physics would predict a match rate as low as 1/4. --Bleckneuhaus  (Diskussion) 04:25, 21. Apr. 2024 (CEST)Beantworten

Noch einfachere Herleitung[Quelltext bearbeiten]

Die Elemente einer Menge können unabhängig voneinander die Eigenschaften besitzen oder nicht besitzen (). Für die Anzahl der Elemente, die sowohl als auch besitzen, kann man mit Hilfe der Eigenschaft eine obere Abschätzung geben. Man teilt in zwei Summanden auf:

Dann ist sicher und . Zusammen also


Ich denke, weder die Aufteilung in 2 Summanden noch die beiden Abschätzungen müssen erläutert werden. Jetzt wäre noch zu hoffen, dass jemand einen Lit-Beleg für die so geschrumpfte Herleitung angeben kann. --Bleckneuhaus (Diskussion) 19:45, 21. Apr. 2024 (CEST)Beantworten

Hallo Bleckneuhaus. In beiden "Herleitungen" tritt die Annahme der Lokalität gar nicht auf. Daher ist mir nicht klar, ob es hier wirklich um die Bellsche Ungleichung geht oder nicht nur um den Beweis, dass man in der QM nicht allen Observablen gleichzeitig konsistent Werte zuweisen kann? Eine Deiner "einfacheren Herleitung" ähnliche Argumentation wird in B. Kümmerer, H. Maassen: Elements of Quantum Probability. In: R. L. Hudson, J. M. Lindsay (Hrsg.): Quantum Probability Communications X. World Scientific, 1998, S. 73–100, doi:10.1142/9789812816054_0003. (vgl. die Proposition Bell's three-variable inequality aus S. 3.) verwendet, um zu zeigen, dass klassische Wahrscheinlichkeitstheorie nicht für die Beschreibung von Quantenexperimenten geeignet ist (jedenfalls nicht ohne aufzupassen, was man als Zufallsvariable ansieht). Aber für die Bell-Ungleichung, die Lokalität und Realismus in Gegensatz zur QM setzt, ist soweit ich sehe, ein weiterer Schritt nötig (vgl. Prop. auf S. 5). --Qcomp (Diskussion) 10:00, 22. Apr. 2024 (CEST)Beantworten
Das habe ich mich auch schon oft gefragt und gedacht, mindestens fehlt ein eigener Abschnitt, der die Verbindung dieser nicht unüblichen Art von (angeblichen) Herleitungen mit den 2-Teilchen-Experimenten erklärt. Ich habe da keinen Überblick über die Literatur und sitze zZt auch ziemlich weitab vom Schuss. - Meine Vermutung ist: das 2. Teilchen wird nur deshalb gebraucht (EPR im Original hatte nur eins!), weil man zwei Eigenschaften desselben (1.) Teilchens bestimmen (und daraus Korrelationen ermitteln) will, wo aber die zweite Eigenschaft einer direkten Messung nicht zugänglich ist, weil sich die Apparaturen für die erste und die zweite Eigenschaft ausschließen. Quantenverschränkung macht das aber doch möglich, und da hat die Lokalität dann auch ihren Platz: die beiden Messungen dürfen sich nicht beeinflussen können. (Hab ich das klar genug ausdrücken können?) --Bleckneuhaus (Diskussion) 19:33, 22. Apr. 2024 (CEST)Beantworten
So kann man es sagen (bzw wird es auch z.B. in dem didaktischen Kümmerer/Maassen-Artikel gesagt): man kann nicht alle diesen gemeinsamen Eigenschaften (a,b,c...) gleichzeitig messen und wenn man es nacheinander tut, aendert sich (zumindest in der QM) die Verteilung. Indem man zwei (geeignet maximal verschränkte) Systeme verwendet, hat man "zwei identische Kopien", an denen man jetzt unabhängig voneinander messen kann. - Das war doch auch schon der Gedanke bei EPR, die ja auch von einem zu 2 Systemen gehen mussten("For this purpose, let us suppose we have two systems,...")
Eiliger Einschub: Danke, dass Du mich endlich von meinem Irrtum befreist, EPR spräche nur von 1 Teilchen! Das hatte ich jahrelang so gedacht, weil Verschränkung hinsichtlich zweier Freiheitsgrade ja auch bei 1 Teilchen vorkommt - zB x- und y-Koordinate bei einer Konzentration von |psi|^2 längs der Diagonalen. So könnte EPR Gl. (9) auch eine 1-Teilchenwellenfunktion sein mit x1 und x2 als xy-Koordinaten. Auf den Kümmerer/Maassen-Artikel kann ich hier leider nicht zugreifen. --Bleckneuhaus (Diskussion) 01:13, 23. Apr. 2024 (CEST)Beantworten
Ein- vs Zwei-Teilchen bei EPR: wie Du schreibst sind ja, was den Hilbertraum angeht, zwei sich im eindimensionalen bewegende Teilchen äquivalent zu einem mit zwei Ortsfreiheitsgraden und man kann den Zustand delta(x1-x2)delta(p1+p2) auf beide Arten interpretieren. Aber bei nir einem Teilchen sehe ich nicht, wie man einen Widerspruch zur Lokalität konstruieren kann. (Das EPR-Argument sollte aber in beiden Fällen durchgehen, wenn man akzeptiert, dass eine Messung der "2"-Komponente von Ort oder Impuls die "1" Komponente (und die durch sie beschriebene physikalische Realität) nicht modifiziert.)
Den Artikel von Kümmerer/Maassen gibt's (soweit ich das feststellen kann: ohne Beschränkung) bei der Radboud University oder ResearchGate. --Qcomp (Diskussion) 12:28, 23. Apr. 2024 (CEST)Beantworten
damit die Messung am zweiten System -jedenfalls bei lokal-realistischer Auffassung- keinen Einfluss auf das nehmen können sollte, was am anderen System gemessen wird.
Wie man umseitig diese Nicht-Bell-Ungleichungen am besten einbringt, ist mir nicht klar. Falls Bell selbst über solche Überlegungen zu seiner Ungleichung gekommen ist, könnte man es in einem Abschnitt zur historischen Entwicklung machen. Ansonsten ginge mE auch unter "Verwandtes". --Qcomp (Diskussion) 00:30, 23. Apr. 2024 (CEST)Beantworten
  1. Einsteins Spuk
  2. John Stewart Bell: On the Einstein Podolsky Rosen Paradox. In: Physics. Band 1, Nr. 3, 1964, S. 195–200 (cern.ch [PDF]).
  3. Friebe, C., Kuhlmann, M., Lyre, H., Näger, P., Passon, O., & Stöckler, M. (2014). Philosophie der Quantenphysik: Einführung und Diskussion der zentralen Begriffe und Problemstellungen der Quantentheorie für Physiker und Philosophen. Springer-Verlag.
  4. Albert Einstein, Boris Podolsky und Nathan Rosen: Can quantum-mechanical description of physical reality be considered complete? In: Phys. Rev. Band 47, 1935, S. 777–780, doi:10.1103/PhysRev.47.777.
  5. Alain Aspect: Bell’s inequality test: more ideal than ever. In: Nature. Band 398, 1999, doi:10.1038/18296.
  6. S. J. Freedman, J. F. Clauser: Experimental Test of Local Hidden-Variable Theories. In: Physical Review Letters. Band 28, Nr. 14, 1972, S. 938–941, doi:10.1103/PhysRevLett.28.938.